LSAT and Law School Admissions Forum

Get expert LSAT preparation and law school admissions advice from PowerScore Test Preparation.

 Administrator
PowerScore Staff
  • PowerScore Staff
  • Posts: 8917
  • Joined: Feb 02, 2011
|
#33112
Complete Question Explanation

Strengthen—PR. The correct answer choice is (E)

The columnist discusses a proposed bill that most people favor. Although the bill does not violate anyone’s basic human rights, the columnist predicts it will not be passed for many years. Additionally, the columnist notes that passage of the bill would have an adverse effect on those who are very influential. The columnist concludes that the delay of the bill shows that this county is not a well-functioning democracy, if it is a democracy at all.

To review, the columnist’s argument, reordered for clarity, proceeds:
  • Premise: ..... Most people favor the bill.

    Premise: ..... The bill does not violate anyone’s basic human rights.

    Premise: ..... Those who would be adversely affected by the bill are very influential.

    Sub-Conclusion: ..... The bill will not be passed for many years, if at all; nor will any other similar bill.

    Conclusion: ..... If this country is a democracy at all, it is not a well-functioning one.
This is a Strengthen—Principle question, in which the correct answer choice will provide a principle, or rule, supporting the columnist’s conclusion that if this country is a democracy at all, it is not a well-functioning one. The conclusion results from the prediction that the bill will not be passed for many years, if at all, nor would any similar bill. The columnist describes the bill as being favored by most people, and doing nothing to violate anyone’s human rights.

Essentially, the columnist is applying a nested conditional rule to the facts: if a bill is favored by most people and does nothing to violate anyone’s human rights, then if the country is a well-functioning democracy, that bill will be made into law before many years go by.
  • Sufficient Necessary

    Bill favored by most

    ..... + ..... ..... ..... :arrow: [well-functioning democracy ..... :arrow: ..... bill passed into law before many years]

    Bill does not violate
    anyone’s human rights

Your prephrase is the correct answer choice will reflect the application of this conditional rule.

Answer choice (A): Though not quite the Mistaken Reversal of the conditional rule applied by the argument, this answer choice results from a similar reversal of the conditions.

Answer choice (B): This answer choice is incorrect because the rule being applied to derive the conclusion does not directly involve the opposition of influential people. Also, this answer choice lacks a statement regarding whether the bill violates anyone’s human rights.

Answer choice (C): This answer choice does not address the impact of the bill on an individual’s human rights, and does not discuss what failure to pass the bill implies regarding the state of democracy in the country.

Answer choice (D): This is the Mistaken Reversal of the rule applied in the stimulus.

Answer choice (E): This is the correct answer choice. As described above, the conclusion results from the application of the nested conditional relationship restated in this answer choice.
 SherryZ
  • Posts: 124
  • Joined: Oct 06, 2013
|
#13115
Oct 2013 LSAT, Sec 1 LR, Q23:

Hi there, could you explain why answer E is right but B is wrong?? I thought B is right :cry:


Thank you and have a nice day ;)

----Sherry
 Ron Gore
PowerScore Staff
  • PowerScore Staff
  • Posts: 220
  • Joined: May 15, 2013
|
#13122
Hi, Sherry!

This is a Strengthen – Principle question, in which the correct answer choice will provide a principle, or rule, supporting the columnist's conclusion that if this country is a democracy at all, it is not a well-functioning one. The conclusion results from the prediction that the bill will not be passed for many years, if at all, nor would any similar bill. The columnist describes the bill as being favored by most people, and doing nothing to violate anyone's human rights.

Essentially, the columnist is applying a nested conditional rule to the facts: if a bill is favored by most people and does nothing to violate anyone's human rights, then if the country is a well-functioning democracy, that bill will be made into law before many years go by.

Answer choice (B) is incorrect for multiple reasons. For one, the argument results in the conclusion that if the country is a democracy at all, it is not a well-functioning one. So, whatever answer choice you select will provide a rule that, like the stimulus, takes the characteristics of the bill as the sufficient condition and infers a conclusion regarding whether the country is a well-functioning democracy. That basic parallel structure is essential.

Also, (B) doesn't reference the effect of the bill on human rights. And, although the stimulus references the opposition of influential people, their influence is not explicitly relied on by the columnist. Rather, the columnist took as evidence the positives of the bill: (1) favored by most people; and (2) does nothing to violate anyone's basic human rights.

Answer choice (E) is correct, because it connects to both of those positives of the bill, creating a rule that results in a conclusion regarding whether the country is a well-functioning democracy. You could diagram (E) as:

Sufficient ..... ..... ..... ..... Necessary

Bill favored by most people

AND ..... ..... :arrow: [well-functioning democracy :arrow: promptly passed into law]

Bill does not violate HR

This certainly was a difficult question. Please let me know if I can help further.

Thanks!

Ron
 maximbasu
  • Posts: 59
  • Joined: May 19, 2016
|
#25142
Hello,
I chose C as the correct answer while the correct answer was D.

I understand that C brings in extraneous information; we are not talking about a bill becoming law--the issue is with the bill NOT becoming law.

I don't understand why E is correct. I'd diagram it as: NOT violate HR --> bill passed.

How does this have anything to do with influential people blocking the legislation?

Thank you, Maxim.
 fg6118
  • Posts: 26
  • Joined: Apr 18, 2016
|
#25174
I have a follow-up question to Ron's diagramming of answer E.

When I read the answer choice, I arranged it as IF the bill does not violate anyone's basic human rights THEN a bill that most people favor will be passed promptly into law in a well functioning democracy.

Bill does not violate anyone's basic human rights --> a bill that most people favor will be passed promptly into law in a well functioning democracy.

Now that I think about it, Ron's diagram makes sense but when you read it, it technically seems right based on the indicator if to diagram it the why I did. How can you double-check the ordering of the your diagram reflects the actual meaning of th statement in spite of what the statement is saying at face value and its indicator words?

Thanks!
 Nikki Siclunov
PowerScore Staff
  • PowerScore Staff
  • Posts: 1362
  • Joined: Aug 02, 2011
|
#25376
Hi Maxim and fg6118,

Your questions are similar, so I thought I'll address both of them at once. Oh, and Maxim - the correct answer choice is (E), not (D), FYI.

It is really, really important not to get "bogged down" in the complex conditional language here and understand the argument holistically. Here's what the columnist essentially says:

The columnist discusses a proposed bill that most people favor. Although the bill does not violate anyone’s basic human rights, the columnist predicts it will not be passed for many years. Additionally, he notes that passage of the bill would have an adverse effect on those who are very influential. From this, the columnist concludes that the delay of the bill shows that this county is not a well-functioning democracy, if it is a democracy at all.

This is a Strengthen—Principle question, in which the correct answer choice will provide a principle (or rule) supporting the columnist’s conclusion that if this country is a democracy at all, it is not a well-functioning one. What we're looking for is something along the following lines:
  • If (1) a bill is favored by most people and (2) it does NOT violate anyone’s human rights, then such a bill would be passed into law in a well-functioning democracy.
This is essentially what answer choice (E) states. The order in which the conditions are presented is a bit different, but the core meaning is the same. Clearly, the bill will not be passed for many years, despite meeting both criteria specified above. It is reasonable to conclude, therefore, that the country is not a well-functioning democracy.

Note that the same idea can be worded (and diagrammed) in several different ways. For instance, we can diagram the statement we are looking for as a Nested Conditional:

Well-functioning democracy :arrow: (Bill favored by most people AND not violate basic rights :arrow: Passed into law)

The necessary condition for a well-functioning democracy is itself a conditional statement, which the example presented in the stimulus clearly violates: the bill will not be passed into law despite meeting both sufficient conditions for such passage. Consequently, if a well-functioning democracy requires such a passage, this country is clearly not a well-functioning democracy.

As you can see, there are various ways of wording the same idea, but if you approach the argument holistically and understand what you're looking for, it would be counterproductive to delve any deeper into the conditional relationships governing the argument structure in this case.

One more thing: the fact that influential people are blocking the legislation is not necessarily something we need to introduce into the argument in order to strengthen the conclusion: this is just one of several characteristics attributable to the bill in question. Maxim - FYI.

Hope this helps clear this up!
 lsat2016
  • Posts: 59
  • Joined: May 29, 2016
|
#25957
Nikki Siclunov wrote:Hi Maxim and fg6118,

One more thing: the fact that influential people are blocking the legislation is not necessarily something we need to introduce into the argument in order to strengthen the conclusion: this is just one of several characteristics attributable to the bill in question. Maxim - FYI.

Hope this helps clear this up!
Hello,

Just from reading the passage alone, how can I know which information is important to include in the information and which is not? For example, could I have justified the statement by introducing the principle:

"bill that adversely affects very influential people not passed" -> not well functioning?
 Nikki Siclunov
PowerScore Staff
  • PowerScore Staff
  • Posts: 1362
  • Joined: Aug 02, 2011
|
#25977
lsat2016,

Good point! :) There are several ways of strengthening the conclusion here, as your example suggests. My point was that it is not necessary to mention each and every characteristic attributable to the bill in question in order to prove the conclusion.

Hope this helps!
 starre
  • Posts: 28
  • Joined: Sep 27, 2016
|
#30597
For this question, I answered A, though I had it narrowed down to A and E. To me, A and E seemed to say the same thing with the difference being the timing of "promptly" vs. "within a few years". I thought "promptly" was too strict of a word, so I went with A. Where did I go wrong?

Thank you in advance!
Emily
 Claire Horan
PowerScore Staff
  • PowerScore Staff
  • Posts: 408
  • Joined: Apr 18, 2016
|
#30661
Hi Emily,

You are right to pinpoint which phrases differ between answer choices. That's a good strategy. For (A) and (E), the most important difference in language was that A refers to a bill that would "benefit" most people, but the stimulus doesn't actually say the bill would benefit most people, only that most people are in favor of it. Answer choice (E) tracks the stimulus better because it describes a bill that most people favor.

Get the most out of your LSAT Prep Plus subscription.

Analyze and track your performance with our Testing and Analytics Package.